Use non-breaking space (~) between 'Figure' and the figure number.
[course.git] / latex / problems / Serway_and_Jewett_8 / problem29.40.tex
1 \begin{problem*}{29.40}
2 Consider the system pictured in Figure~P29.40.  A $15.0\U{cm}$
3 horizontal wire of mass $15.0\U{g}$ is placed between two thin,
4 vertical conductors, and a uniform magnetic field acts perpendicular
5 to the page.  The wire is free to move vertically without friction on
6 the two vertical conductors.  When a $5.00\U{A}$ curent is directed as
7 shown in the figure, the horizontal wire moves upward at a constant
8 velocity in the presence of gravity.  \Part{a} What forces act on the
9 horizontal wire, and \Part{b} under what condition is the wire able to
10 move upward at a constant velocity.  \Part{c} Find the magnitude and
11 direction of the minimum magnetic field required to move the wire at a
12 constant speed.  \Part{d} What happens if the magnetic field exceeds
13 this minimum value?
14 \begin{center}
15 % |     |  15.0 cm between verticals
16 % +--<--+
17 % |  5A |
18 % |     |
19 % v 5A  ^ 5A
20 \begin{asy}
21 import Mechanics;
22 import Circ;
23
24 real u = 1cm;
25
26 pair ul = (-u, u);    // upper left
27 pair ml = (-u, 0);    // middle left
28 pair ll = (-u, -2u);  // lower left
29 pair ur = (u, u);     // upper right
30 pair mr = (u, 0);
31 pair lr = (u, -2u);
32
33 MultiTerminal I = current(label=Label("$5.00\U{A}$"), draw=false);
34 I.centerto(lr, mr);  I.label.align = E;  I.draw();
35 wire(lr, I.terminal[0]);
36 wire(I.terminal[1], ur);
37 I.centerto(mr, ml);  I.label.align = S;  I.draw();
38 wire(mr, I.terminal[0]);
39 wire(I.terminal[1], ml);
40 I.centerto(ml, ll);  I.label.align = W;  I.draw();
41 wire(ul, I.terminal[0]);
42 wire(I.terminal[1], ll);
43 dot(mr);
44 dot(ml);
45
46 Distance D = Distance(ml, mr, offset=-u/2, Label("$15.0\U{cm}$", align=N));
47 D.draw();
48 \end{asy}
49 \end{center}
50 \end{problem*}
51
52 \begin{solution}
53 \Part{a}
54 The only forces acting on the horizontal wire are \ans{gravity and the
55 force of the magnetic field on the current carried by the wire}.
56
57 \Part{b}
58 If the wire is moving at a constant velocity,
59 \begin{align}
60   0 &= \deriv{t}{\vect{v}} = \vect{a} = \frac{\vect{F}}{m} \\
61   \ans{\vect{F}} &= \ans{0} \;.
62 \end{align}
63 In English, this means the net force on the wire must be zero.
64 Because there are only two forces acting on the wire, this means these
65 forces must be equal in magnitude and opposite in direction.
66
67 \Part{c}
68 The gravitational force on the wire will be $F_g=mg$ directed
69 downwards, and the magnetic force on the wire will be
70 $F_B=I\vect{L}\times\vect{B}$ directed upwards.  Because the magnetic
71 field is perpendicular to the page, and the current is directed along
72 the page, the angle between $\vect{L}$ and $\vect{B}$ is $90\dg$, so
73 $F_B=ILB\sin{90\dg}=ILB$.  Summing forces in the vertical direction,
74 \begin{align}
75   0 &= \sum_i F_{i,y} = F_B - F_g = ILB - mg \\
76   ILB &= mg \\
77   B &= \frac{mg}{IL}
78     = \frac{0.0150\U{kg} \cdot 9.80\U{m/s$^2$}}{5.00\U{A} \cdot 0.150\U{m}}
79     = \ans{196\U{mT}} \;.
80 \end{align}
81 In order to generate an upwards force on a leftwards current, the
82 magnetic field must be directed \ans{out of the page}.
83
84 \Part{d}
85 If the magnetic field exceeds this value, \ans{the rod will accelerate
86   upwards}, moving up faster and faster.  If the magnetic field is
87 less than this value, the rod will accelerate downwards, moving up
88 slower and slower, eventually pausing, and then moving back down
89 faster and faster.  At the critical magnetic field, where the forces
90 balance, the rod will keep moving forever at it's initial velocity (or
91 hang in place if the initial velocity was zero).
92 \end{solution}